Find of two arithmetic progressions meet












0














I was working a problem to find if two AP meet or not and came across the below post
Formula to find if two AP meet



and in that the formula given by user is



$$A_n=A_1+(n−1)d$$



$$B_m=B_1+(m−1)D$$



and if $A_n=B_m$ then they meet. But somehow it failed for my case of $A_1=14,d=4 B_1=98,D=2$.
Could you please tell me where i went wrong or the formula was wrong?










share|cite|improve this question









New contributor




hardyz009 is a new contributor to this site. Take care in asking for clarification, commenting, and answering.
Check out our Code of Conduct.




















  • Welcome to Math SE. By definition, the formula is right. In your failure case, what are the values of $n$ for $A_n$ and $m$ for $B_m$ that you tried using?
    – John Omielan
    3 hours ago












  • so as per the explanation given by user i got n as 2 and m as 1.
    – hardyz009
    3 hours ago










  • Thanks for providing those 2 values. Based on your provided values, the $2$ arithmetic progressions don't meet at that point, as you determined yourself.
    – John Omielan
    3 hours ago












  • as per the formula they dont but the actual answer is they do meet and i am trying to figure out where did i go wrong
    – hardyz009
    3 hours ago










  • they both meet at 162.
    – hardyz009
    3 hours ago
















0














I was working a problem to find if two AP meet or not and came across the below post
Formula to find if two AP meet



and in that the formula given by user is



$$A_n=A_1+(n−1)d$$



$$B_m=B_1+(m−1)D$$



and if $A_n=B_m$ then they meet. But somehow it failed for my case of $A_1=14,d=4 B_1=98,D=2$.
Could you please tell me where i went wrong or the formula was wrong?










share|cite|improve this question









New contributor




hardyz009 is a new contributor to this site. Take care in asking for clarification, commenting, and answering.
Check out our Code of Conduct.




















  • Welcome to Math SE. By definition, the formula is right. In your failure case, what are the values of $n$ for $A_n$ and $m$ for $B_m$ that you tried using?
    – John Omielan
    3 hours ago












  • so as per the explanation given by user i got n as 2 and m as 1.
    – hardyz009
    3 hours ago










  • Thanks for providing those 2 values. Based on your provided values, the $2$ arithmetic progressions don't meet at that point, as you determined yourself.
    – John Omielan
    3 hours ago












  • as per the formula they dont but the actual answer is they do meet and i am trying to figure out where did i go wrong
    – hardyz009
    3 hours ago










  • they both meet at 162.
    – hardyz009
    3 hours ago














0












0








0







I was working a problem to find if two AP meet or not and came across the below post
Formula to find if two AP meet



and in that the formula given by user is



$$A_n=A_1+(n−1)d$$



$$B_m=B_1+(m−1)D$$



and if $A_n=B_m$ then they meet. But somehow it failed for my case of $A_1=14,d=4 B_1=98,D=2$.
Could you please tell me where i went wrong or the formula was wrong?










share|cite|improve this question









New contributor




hardyz009 is a new contributor to this site. Take care in asking for clarification, commenting, and answering.
Check out our Code of Conduct.











I was working a problem to find if two AP meet or not and came across the below post
Formula to find if two AP meet



and in that the formula given by user is



$$A_n=A_1+(n−1)d$$



$$B_m=B_1+(m−1)D$$



and if $A_n=B_m$ then they meet. But somehow it failed for my case of $A_1=14,d=4 B_1=98,D=2$.
Could you please tell me where i went wrong or the formula was wrong?







sequences-and-series arithmetic arithmetic-progressions






share|cite|improve this question









New contributor




hardyz009 is a new contributor to this site. Take care in asking for clarification, commenting, and answering.
Check out our Code of Conduct.











share|cite|improve this question









New contributor




hardyz009 is a new contributor to this site. Take care in asking for clarification, commenting, and answering.
Check out our Code of Conduct.









share|cite|improve this question




share|cite|improve this question








edited 3 hours ago









user26857

39.2k123882




39.2k123882






New contributor




hardyz009 is a new contributor to this site. Take care in asking for clarification, commenting, and answering.
Check out our Code of Conduct.









asked 3 hours ago









hardyz009

12




12




New contributor




hardyz009 is a new contributor to this site. Take care in asking for clarification, commenting, and answering.
Check out our Code of Conduct.





New contributor





hardyz009 is a new contributor to this site. Take care in asking for clarification, commenting, and answering.
Check out our Code of Conduct.






hardyz009 is a new contributor to this site. Take care in asking for clarification, commenting, and answering.
Check out our Code of Conduct.












  • Welcome to Math SE. By definition, the formula is right. In your failure case, what are the values of $n$ for $A_n$ and $m$ for $B_m$ that you tried using?
    – John Omielan
    3 hours ago












  • so as per the explanation given by user i got n as 2 and m as 1.
    – hardyz009
    3 hours ago










  • Thanks for providing those 2 values. Based on your provided values, the $2$ arithmetic progressions don't meet at that point, as you determined yourself.
    – John Omielan
    3 hours ago












  • as per the formula they dont but the actual answer is they do meet and i am trying to figure out where did i go wrong
    – hardyz009
    3 hours ago










  • they both meet at 162.
    – hardyz009
    3 hours ago


















  • Welcome to Math SE. By definition, the formula is right. In your failure case, what are the values of $n$ for $A_n$ and $m$ for $B_m$ that you tried using?
    – John Omielan
    3 hours ago












  • so as per the explanation given by user i got n as 2 and m as 1.
    – hardyz009
    3 hours ago










  • Thanks for providing those 2 values. Based on your provided values, the $2$ arithmetic progressions don't meet at that point, as you determined yourself.
    – John Omielan
    3 hours ago












  • as per the formula they dont but the actual answer is they do meet and i am trying to figure out where did i go wrong
    – hardyz009
    3 hours ago










  • they both meet at 162.
    – hardyz009
    3 hours ago
















Welcome to Math SE. By definition, the formula is right. In your failure case, what are the values of $n$ for $A_n$ and $m$ for $B_m$ that you tried using?
– John Omielan
3 hours ago






Welcome to Math SE. By definition, the formula is right. In your failure case, what are the values of $n$ for $A_n$ and $m$ for $B_m$ that you tried using?
– John Omielan
3 hours ago














so as per the explanation given by user i got n as 2 and m as 1.
– hardyz009
3 hours ago




so as per the explanation given by user i got n as 2 and m as 1.
– hardyz009
3 hours ago












Thanks for providing those 2 values. Based on your provided values, the $2$ arithmetic progressions don't meet at that point, as you determined yourself.
– John Omielan
3 hours ago






Thanks for providing those 2 values. Based on your provided values, the $2$ arithmetic progressions don't meet at that point, as you determined yourself.
– John Omielan
3 hours ago














as per the formula they dont but the actual answer is they do meet and i am trying to figure out where did i go wrong
– hardyz009
3 hours ago




as per the formula they dont but the actual answer is they do meet and i am trying to figure out where did i go wrong
– hardyz009
3 hours ago












they both meet at 162.
– hardyz009
3 hours ago




they both meet at 162.
– hardyz009
3 hours ago










1 Answer
1






active

oldest

votes


















0














The $2$ sequences meet, but with $n$ being $22$, not $2$, for $m$ being $1$, with them meeting at $98$, as can be easily confirmed by plugging the values into the equations:
$$14 + left(22 - 1right)4 = 14 + left(21right)4 = 14 + 84 = 98$$
$$98 + left(1 - 1right)2 = 98 + 0 = 98$$
Note to determine $n$ and $m$, subtracting the second general formula from the first general one provided in the question gives:
$$84 = left(n - 1right)4 - left(m - 1right)2$$
$$84 = 4n - 4 - 2m + 2$$
$$86 = 4n - 2m$$
$$43 = 2n - m$$
Since both $m$ and $n$ must be $geq 1$, so this limits what there values can be. As $43$ is odd and $2n$ is even, the values of $m$ must all be odd. The smallest odd value is $1$, with this giving $2n = 43 + 1 = 44$, so $n = 22$.

Since $84$ is a multiple of $4$, the values which work is that $n - 1 = 84/4 = 21$, so $n = 22$, with this giving $m = 1$. For each increase in $n$, the right side increases by $2$, so $m$ must increase by $2$ (as it is being subtracted) to compensate to keep it an equation. Thus, the sets of $left(n,mright)$ values which work are $left(22,1right),left(23,3right),left(24,5right),ldots$, with this going up to $left(38,33right)$ which gives $162$, as stated by the question's author, hardyz009, in his/her comment to the original question.






share|cite|improve this answer























  • how did u compute value of n and m.Because as per the explanation we compute n and m by dividing d and D with the gcd. Hence n=4/2 and m=2/2
    – hardyz009
    3 hours ago












  • $hardz009, I have added a few more details to my answer, so I hope it explains it better for you and that you now understand it.
    – John Omielan
    3 hours ago












  • when you get the equation 84=(m−1)2−(n−1)4 but to calculate we just consider 9n-1)=84 shouldnt the above equation be resolved to 2n-m=41, because we just cant break the equation in between to find values of n and m.
    – hardyz009
    2 hours ago










  • @hardz009 Yes, the equation can be simplified to $43 = 2n - m$. However, note that $n$ and $m$ need to each be $ge 1$, so this limits what values they can take. Since 43 is an odd number and $2n$ is even, this means that $m$ must be odd, with the 1st value being 1. This gives that $n = 22$. For each increase in $n$, the right side increases by 2, so $m$ must increase by 2 (as it is being subtracted) to compensate to keep it an equation. Thus, the sets of $left(n, mright)$ values which work are $left(22, 1right), left(23, 3right), left(24, 5right), ldots$.
    – John Omielan
    2 hours ago










  • but then there has to be a way of calculating n and m with 43=2n-m the point you stated is valid but if it would have been a bigger number or lhs had been even then we wouldnt be doing a hit and trial to compute n and m.
    – hardyz009
    2 hours ago











Your Answer





StackExchange.ifUsing("editor", function () {
return StackExchange.using("mathjaxEditing", function () {
StackExchange.MarkdownEditor.creationCallbacks.add(function (editor, postfix) {
StackExchange.mathjaxEditing.prepareWmdForMathJax(editor, postfix, [["$", "$"], ["\\(","\\)"]]);
});
});
}, "mathjax-editing");

StackExchange.ready(function() {
var channelOptions = {
tags: "".split(" "),
id: "69"
};
initTagRenderer("".split(" "), "".split(" "), channelOptions);

StackExchange.using("externalEditor", function() {
// Have to fire editor after snippets, if snippets enabled
if (StackExchange.settings.snippets.snippetsEnabled) {
StackExchange.using("snippets", function() {
createEditor();
});
}
else {
createEditor();
}
});

function createEditor() {
StackExchange.prepareEditor({
heartbeatType: 'answer',
autoActivateHeartbeat: false,
convertImagesToLinks: true,
noModals: true,
showLowRepImageUploadWarning: true,
reputationToPostImages: 10,
bindNavPrevention: true,
postfix: "",
imageUploader: {
brandingHtml: "Powered by u003ca class="icon-imgur-white" href="https://imgur.com/"u003eu003c/au003e",
contentPolicyHtml: "User contributions licensed under u003ca href="https://creativecommons.org/licenses/by-sa/3.0/"u003ecc by-sa 3.0 with attribution requiredu003c/au003e u003ca href="https://stackoverflow.com/legal/content-policy"u003e(content policy)u003c/au003e",
allowUrls: true
},
noCode: true, onDemand: true,
discardSelector: ".discard-answer"
,immediatelyShowMarkdownHelp:true
});


}
});






hardyz009 is a new contributor. Be nice, and check out our Code of Conduct.










draft saved

draft discarded


















StackExchange.ready(
function () {
StackExchange.openid.initPostLogin('.new-post-login', 'https%3a%2f%2fmath.stackexchange.com%2fquestions%2f3051893%2ffind-of-two-arithmetic-progressions-meet%23new-answer', 'question_page');
}
);

Post as a guest















Required, but never shown

























1 Answer
1






active

oldest

votes








1 Answer
1






active

oldest

votes









active

oldest

votes






active

oldest

votes









0














The $2$ sequences meet, but with $n$ being $22$, not $2$, for $m$ being $1$, with them meeting at $98$, as can be easily confirmed by plugging the values into the equations:
$$14 + left(22 - 1right)4 = 14 + left(21right)4 = 14 + 84 = 98$$
$$98 + left(1 - 1right)2 = 98 + 0 = 98$$
Note to determine $n$ and $m$, subtracting the second general formula from the first general one provided in the question gives:
$$84 = left(n - 1right)4 - left(m - 1right)2$$
$$84 = 4n - 4 - 2m + 2$$
$$86 = 4n - 2m$$
$$43 = 2n - m$$
Since both $m$ and $n$ must be $geq 1$, so this limits what there values can be. As $43$ is odd and $2n$ is even, the values of $m$ must all be odd. The smallest odd value is $1$, with this giving $2n = 43 + 1 = 44$, so $n = 22$.

Since $84$ is a multiple of $4$, the values which work is that $n - 1 = 84/4 = 21$, so $n = 22$, with this giving $m = 1$. For each increase in $n$, the right side increases by $2$, so $m$ must increase by $2$ (as it is being subtracted) to compensate to keep it an equation. Thus, the sets of $left(n,mright)$ values which work are $left(22,1right),left(23,3right),left(24,5right),ldots$, with this going up to $left(38,33right)$ which gives $162$, as stated by the question's author, hardyz009, in his/her comment to the original question.






share|cite|improve this answer























  • how did u compute value of n and m.Because as per the explanation we compute n and m by dividing d and D with the gcd. Hence n=4/2 and m=2/2
    – hardyz009
    3 hours ago












  • $hardz009, I have added a few more details to my answer, so I hope it explains it better for you and that you now understand it.
    – John Omielan
    3 hours ago












  • when you get the equation 84=(m−1)2−(n−1)4 but to calculate we just consider 9n-1)=84 shouldnt the above equation be resolved to 2n-m=41, because we just cant break the equation in between to find values of n and m.
    – hardyz009
    2 hours ago










  • @hardz009 Yes, the equation can be simplified to $43 = 2n - m$. However, note that $n$ and $m$ need to each be $ge 1$, so this limits what values they can take. Since 43 is an odd number and $2n$ is even, this means that $m$ must be odd, with the 1st value being 1. This gives that $n = 22$. For each increase in $n$, the right side increases by 2, so $m$ must increase by 2 (as it is being subtracted) to compensate to keep it an equation. Thus, the sets of $left(n, mright)$ values which work are $left(22, 1right), left(23, 3right), left(24, 5right), ldots$.
    – John Omielan
    2 hours ago










  • but then there has to be a way of calculating n and m with 43=2n-m the point you stated is valid but if it would have been a bigger number or lhs had been even then we wouldnt be doing a hit and trial to compute n and m.
    – hardyz009
    2 hours ago
















0














The $2$ sequences meet, but with $n$ being $22$, not $2$, for $m$ being $1$, with them meeting at $98$, as can be easily confirmed by plugging the values into the equations:
$$14 + left(22 - 1right)4 = 14 + left(21right)4 = 14 + 84 = 98$$
$$98 + left(1 - 1right)2 = 98 + 0 = 98$$
Note to determine $n$ and $m$, subtracting the second general formula from the first general one provided in the question gives:
$$84 = left(n - 1right)4 - left(m - 1right)2$$
$$84 = 4n - 4 - 2m + 2$$
$$86 = 4n - 2m$$
$$43 = 2n - m$$
Since both $m$ and $n$ must be $geq 1$, so this limits what there values can be. As $43$ is odd and $2n$ is even, the values of $m$ must all be odd. The smallest odd value is $1$, with this giving $2n = 43 + 1 = 44$, so $n = 22$.

Since $84$ is a multiple of $4$, the values which work is that $n - 1 = 84/4 = 21$, so $n = 22$, with this giving $m = 1$. For each increase in $n$, the right side increases by $2$, so $m$ must increase by $2$ (as it is being subtracted) to compensate to keep it an equation. Thus, the sets of $left(n,mright)$ values which work are $left(22,1right),left(23,3right),left(24,5right),ldots$, with this going up to $left(38,33right)$ which gives $162$, as stated by the question's author, hardyz009, in his/her comment to the original question.






share|cite|improve this answer























  • how did u compute value of n and m.Because as per the explanation we compute n and m by dividing d and D with the gcd. Hence n=4/2 and m=2/2
    – hardyz009
    3 hours ago












  • $hardz009, I have added a few more details to my answer, so I hope it explains it better for you and that you now understand it.
    – John Omielan
    3 hours ago












  • when you get the equation 84=(m−1)2−(n−1)4 but to calculate we just consider 9n-1)=84 shouldnt the above equation be resolved to 2n-m=41, because we just cant break the equation in between to find values of n and m.
    – hardyz009
    2 hours ago










  • @hardz009 Yes, the equation can be simplified to $43 = 2n - m$. However, note that $n$ and $m$ need to each be $ge 1$, so this limits what values they can take. Since 43 is an odd number and $2n$ is even, this means that $m$ must be odd, with the 1st value being 1. This gives that $n = 22$. For each increase in $n$, the right side increases by 2, so $m$ must increase by 2 (as it is being subtracted) to compensate to keep it an equation. Thus, the sets of $left(n, mright)$ values which work are $left(22, 1right), left(23, 3right), left(24, 5right), ldots$.
    – John Omielan
    2 hours ago










  • but then there has to be a way of calculating n and m with 43=2n-m the point you stated is valid but if it would have been a bigger number or lhs had been even then we wouldnt be doing a hit and trial to compute n and m.
    – hardyz009
    2 hours ago














0












0








0






The $2$ sequences meet, but with $n$ being $22$, not $2$, for $m$ being $1$, with them meeting at $98$, as can be easily confirmed by plugging the values into the equations:
$$14 + left(22 - 1right)4 = 14 + left(21right)4 = 14 + 84 = 98$$
$$98 + left(1 - 1right)2 = 98 + 0 = 98$$
Note to determine $n$ and $m$, subtracting the second general formula from the first general one provided in the question gives:
$$84 = left(n - 1right)4 - left(m - 1right)2$$
$$84 = 4n - 4 - 2m + 2$$
$$86 = 4n - 2m$$
$$43 = 2n - m$$
Since both $m$ and $n$ must be $geq 1$, so this limits what there values can be. As $43$ is odd and $2n$ is even, the values of $m$ must all be odd. The smallest odd value is $1$, with this giving $2n = 43 + 1 = 44$, so $n = 22$.

Since $84$ is a multiple of $4$, the values which work is that $n - 1 = 84/4 = 21$, so $n = 22$, with this giving $m = 1$. For each increase in $n$, the right side increases by $2$, so $m$ must increase by $2$ (as it is being subtracted) to compensate to keep it an equation. Thus, the sets of $left(n,mright)$ values which work are $left(22,1right),left(23,3right),left(24,5right),ldots$, with this going up to $left(38,33right)$ which gives $162$, as stated by the question's author, hardyz009, in his/her comment to the original question.






share|cite|improve this answer














The $2$ sequences meet, but with $n$ being $22$, not $2$, for $m$ being $1$, with them meeting at $98$, as can be easily confirmed by plugging the values into the equations:
$$14 + left(22 - 1right)4 = 14 + left(21right)4 = 14 + 84 = 98$$
$$98 + left(1 - 1right)2 = 98 + 0 = 98$$
Note to determine $n$ and $m$, subtracting the second general formula from the first general one provided in the question gives:
$$84 = left(n - 1right)4 - left(m - 1right)2$$
$$84 = 4n - 4 - 2m + 2$$
$$86 = 4n - 2m$$
$$43 = 2n - m$$
Since both $m$ and $n$ must be $geq 1$, so this limits what there values can be. As $43$ is odd and $2n$ is even, the values of $m$ must all be odd. The smallest odd value is $1$, with this giving $2n = 43 + 1 = 44$, so $n = 22$.

Since $84$ is a multiple of $4$, the values which work is that $n - 1 = 84/4 = 21$, so $n = 22$, with this giving $m = 1$. For each increase in $n$, the right side increases by $2$, so $m$ must increase by $2$ (as it is being subtracted) to compensate to keep it an equation. Thus, the sets of $left(n,mright)$ values which work are $left(22,1right),left(23,3right),left(24,5right),ldots$, with this going up to $left(38,33right)$ which gives $162$, as stated by the question's author, hardyz009, in his/her comment to the original question.







share|cite|improve this answer














share|cite|improve this answer



share|cite|improve this answer








edited 1 hour ago

























answered 3 hours ago









John Omielan

3317




3317












  • how did u compute value of n and m.Because as per the explanation we compute n and m by dividing d and D with the gcd. Hence n=4/2 and m=2/2
    – hardyz009
    3 hours ago












  • $hardz009, I have added a few more details to my answer, so I hope it explains it better for you and that you now understand it.
    – John Omielan
    3 hours ago












  • when you get the equation 84=(m−1)2−(n−1)4 but to calculate we just consider 9n-1)=84 shouldnt the above equation be resolved to 2n-m=41, because we just cant break the equation in between to find values of n and m.
    – hardyz009
    2 hours ago










  • @hardz009 Yes, the equation can be simplified to $43 = 2n - m$. However, note that $n$ and $m$ need to each be $ge 1$, so this limits what values they can take. Since 43 is an odd number and $2n$ is even, this means that $m$ must be odd, with the 1st value being 1. This gives that $n = 22$. For each increase in $n$, the right side increases by 2, so $m$ must increase by 2 (as it is being subtracted) to compensate to keep it an equation. Thus, the sets of $left(n, mright)$ values which work are $left(22, 1right), left(23, 3right), left(24, 5right), ldots$.
    – John Omielan
    2 hours ago










  • but then there has to be a way of calculating n and m with 43=2n-m the point you stated is valid but if it would have been a bigger number or lhs had been even then we wouldnt be doing a hit and trial to compute n and m.
    – hardyz009
    2 hours ago


















  • how did u compute value of n and m.Because as per the explanation we compute n and m by dividing d and D with the gcd. Hence n=4/2 and m=2/2
    – hardyz009
    3 hours ago












  • $hardz009, I have added a few more details to my answer, so I hope it explains it better for you and that you now understand it.
    – John Omielan
    3 hours ago












  • when you get the equation 84=(m−1)2−(n−1)4 but to calculate we just consider 9n-1)=84 shouldnt the above equation be resolved to 2n-m=41, because we just cant break the equation in between to find values of n and m.
    – hardyz009
    2 hours ago










  • @hardz009 Yes, the equation can be simplified to $43 = 2n - m$. However, note that $n$ and $m$ need to each be $ge 1$, so this limits what values they can take. Since 43 is an odd number and $2n$ is even, this means that $m$ must be odd, with the 1st value being 1. This gives that $n = 22$. For each increase in $n$, the right side increases by 2, so $m$ must increase by 2 (as it is being subtracted) to compensate to keep it an equation. Thus, the sets of $left(n, mright)$ values which work are $left(22, 1right), left(23, 3right), left(24, 5right), ldots$.
    – John Omielan
    2 hours ago










  • but then there has to be a way of calculating n and m with 43=2n-m the point you stated is valid but if it would have been a bigger number or lhs had been even then we wouldnt be doing a hit and trial to compute n and m.
    – hardyz009
    2 hours ago
















how did u compute value of n and m.Because as per the explanation we compute n and m by dividing d and D with the gcd. Hence n=4/2 and m=2/2
– hardyz009
3 hours ago






how did u compute value of n and m.Because as per the explanation we compute n and m by dividing d and D with the gcd. Hence n=4/2 and m=2/2
– hardyz009
3 hours ago














$hardz009, I have added a few more details to my answer, so I hope it explains it better for you and that you now understand it.
– John Omielan
3 hours ago






$hardz009, I have added a few more details to my answer, so I hope it explains it better for you and that you now understand it.
– John Omielan
3 hours ago














when you get the equation 84=(m−1)2−(n−1)4 but to calculate we just consider 9n-1)=84 shouldnt the above equation be resolved to 2n-m=41, because we just cant break the equation in between to find values of n and m.
– hardyz009
2 hours ago




when you get the equation 84=(m−1)2−(n−1)4 but to calculate we just consider 9n-1)=84 shouldnt the above equation be resolved to 2n-m=41, because we just cant break the equation in between to find values of n and m.
– hardyz009
2 hours ago












@hardz009 Yes, the equation can be simplified to $43 = 2n - m$. However, note that $n$ and $m$ need to each be $ge 1$, so this limits what values they can take. Since 43 is an odd number and $2n$ is even, this means that $m$ must be odd, with the 1st value being 1. This gives that $n = 22$. For each increase in $n$, the right side increases by 2, so $m$ must increase by 2 (as it is being subtracted) to compensate to keep it an equation. Thus, the sets of $left(n, mright)$ values which work are $left(22, 1right), left(23, 3right), left(24, 5right), ldots$.
– John Omielan
2 hours ago




@hardz009 Yes, the equation can be simplified to $43 = 2n - m$. However, note that $n$ and $m$ need to each be $ge 1$, so this limits what values they can take. Since 43 is an odd number and $2n$ is even, this means that $m$ must be odd, with the 1st value being 1. This gives that $n = 22$. For each increase in $n$, the right side increases by 2, so $m$ must increase by 2 (as it is being subtracted) to compensate to keep it an equation. Thus, the sets of $left(n, mright)$ values which work are $left(22, 1right), left(23, 3right), left(24, 5right), ldots$.
– John Omielan
2 hours ago












but then there has to be a way of calculating n and m with 43=2n-m the point you stated is valid but if it would have been a bigger number or lhs had been even then we wouldnt be doing a hit and trial to compute n and m.
– hardyz009
2 hours ago




but then there has to be a way of calculating n and m with 43=2n-m the point you stated is valid but if it would have been a bigger number or lhs had been even then we wouldnt be doing a hit and trial to compute n and m.
– hardyz009
2 hours ago










hardyz009 is a new contributor. Be nice, and check out our Code of Conduct.










draft saved

draft discarded


















hardyz009 is a new contributor. Be nice, and check out our Code of Conduct.













hardyz009 is a new contributor. Be nice, and check out our Code of Conduct.












hardyz009 is a new contributor. Be nice, and check out our Code of Conduct.
















Thanks for contributing an answer to Mathematics Stack Exchange!


  • Please be sure to answer the question. Provide details and share your research!

But avoid



  • Asking for help, clarification, or responding to other answers.

  • Making statements based on opinion; back them up with references or personal experience.


Use MathJax to format equations. MathJax reference.


To learn more, see our tips on writing great answers.





Some of your past answers have not been well-received, and you're in danger of being blocked from answering.


Please pay close attention to the following guidance:


  • Please be sure to answer the question. Provide details and share your research!

But avoid



  • Asking for help, clarification, or responding to other answers.

  • Making statements based on opinion; back them up with references or personal experience.


To learn more, see our tips on writing great answers.




draft saved


draft discarded














StackExchange.ready(
function () {
StackExchange.openid.initPostLogin('.new-post-login', 'https%3a%2f%2fmath.stackexchange.com%2fquestions%2f3051893%2ffind-of-two-arithmetic-progressions-meet%23new-answer', 'question_page');
}
);

Post as a guest















Required, but never shown





















































Required, but never shown














Required, but never shown












Required, but never shown







Required, but never shown

































Required, but never shown














Required, but never shown












Required, but never shown







Required, but never shown







Popular posts from this blog

Human spaceflight

Can not write log (Is /dev/pts mounted?) - openpty in Ubuntu-on-Windows?

File:DeusFollowingSea.jpg